what is the proportion 4/y= 5/10

Answers

Answer 1

Answer:

So, y =8

Step-by-step explanation:

4/y = 5/10

or, 4*10 = 5*y

or, 40 = 5y

or, y = 40/5

therefore, y = 8


Related Questions

pls help me don't know what to do

Answers

Answer:

x=15

Step-by-step explanation:

The 60 degree angle and the (x+45) degree angle are both the same degree because they are vertical angles.

So to solve, just subtract 45 from 60

60-45=15

That's your answer!

Hope this helps!

There are 35 times as many students at Wow University as teachers. When all the students and
teachers are seated in the 8544 seat auditorium, 12 seats are empty. How many students attend
Wow University?

Answers

Given:

There are 35 times as many students at Wow University as teachers.

When all the students and teachers are seated in the 8544 seat auditorium, 12 seats are empty.

To find:

The total number of students.

Solution:

Let x be the number of teachers at Wow University. So, the number of student is :

[tex]35\times x=35x[/tex]

When all the students and teachers are seated in the 8544 seat auditorium, 12 seats are empty.

[tex]x+35x=8544-12[/tex]

[tex]36x=8532[/tex]

[tex]x=\dfrac{8532}{36}[/tex]

[tex]x=237[/tex]

The number of total students is:

[tex]35x=35(237)[/tex]

[tex]35x=8295[/tex]

Therefore, the total number of students is 8295.

Arturo is building a flower bed in the shape of a right triangle. The hypotenuse of the right triangle is 13 feet. One of the legs needs to be 7 feet longer than the other leg.

Which equation can be used to find x, the length of the shorter leg?

x(x + 7) = 13
x(x + 7) = 169
x2 + (x + 7)2 = 13
x2 + (x + 7)2 = 169

Answers

Answer:

x(x + 7) = 169

Step-by-step explanation:

We are given that

Let length of shorter leg of right triangle = x ft

Length of other leg = x+7 ft

Hypotenuse of right triangle = 13 ft

We have to find the equation which can be used to find x, the length of shorter leg.

By pythagorous theorem

(Hypotenuse)^2=(Base)^2+(Perpendicular\;side)^2

Substitute the values then, we get

(13)^2=x^2+(x+7)^2

x^2+(x+7)^2=169

This is required equation which can used to find x.

Answer:

d. x2 + (x + 7)2 = 169

Step-by-step explanation:

use Pythagorean theorem, substitute in the given values, and you get this

D=22/7×d-90 Solve the equation
Find D​
Fast!

Answers

Answer:

D=22-90+22d/7

Step-by-step explanation:

D=22/7×d-90

D=-90+22d/7

Answer is- d=-630/-22x+7

A statistics professor asked students in a class their ages. On the basis of this information, the professor states that the average age of all the students in the university is 24 years. This is an example of

Answers

Answer:

propbability ???

Step-bp explanation:

Answer:

Step-by-step explanation:

This is an example of a statistical mean.

Find ∠MPN

Help me please

Answers

Answer:

[tex]22^{\circ}[/tex]

Step-by-step explanation:

Line [tex]\overline{PM}[/tex] is a diameter of the circle because it passes through the circle's center O. Therefore, arc [tex]\widehat{PLM}[/tex] must be 180 degrees, as these are 360 degree in a circle.

We can then find the measure of arc [tex]\widehat{LM}[/tex]:

[tex]\widehat{LP}+\widehat{LM}=180^{\circ},\\92^{\circ}+\widehat{LM}=180^{\circ},\\\widehat{LM}=88^{\circ}[/tex]

Arc [tex]\widehat{LM}[/tex] is formed by angle [tex]\angle LPM[/tex]. Define an inscribed angle by an angle with a point on the circle creating an arc on the circumference of the circle. The measure of an inscribed angle is exactly half of the measure of the arc it forms.

Therefore, the measure of [tex]\angle LPM[/tex] must be:

[tex]m\angle LPM=\frac{88}{2}=44^{\circ}[/tex]

Similarly, the measure of [tex]\angle LNP[/tex] must be:

[tex]m\angle LNP=\frac{92}{2}=46^{\circ}[/tex]

Angles [tex]\angle LPM[/tex] and [tex]\angle MPN[/tex] form angle [tex]\angle LPN[/tex], which is one of the three angles in [tex]\triangle LPN[/tex]. Since the sum of the interior angles of a triangle add up to 180 degrees, we have:

[tex](\angle MPN+\angle LPM)+\angl+ PLN+\angle LNP=180^{\circ},\\\angle MPN+44+46+68=180,\\\angle MPN=180-44-46-68,\\\angle MPN=\boxed{22^{\circ}}[/tex]

please I need answer right now ppleassssee

D. -1/4+-2/3
with explanation pleaseeee​

Answers

Answer:

1 - 1/3

Step-by-step explanation:

Since 1/2 * 2/3 = 1/3, it should be the case that 1/3 divided by 2/3 gives 1/2 and that 1/3 divided by 1/2 gives 2/3. To divide fractions, we multiply the numerator by the reciprocal of the denominator, where the reciprocal of a number just interchanges the numerator and denominator of the number.

What is the quotient of (x^3 - 3x^2 + 3x - 2) ÷ (x^2 - x + 1)?
O x - 2
O x + 2
O x- 4
O x + 1

Answers

Answer:

x-2

The choose (1)

Step-by-step explanation:

(x³-3x²+3x-2)÷(x²-x+1)

(x-2)(x²-x+1) ÷ (x²-x+1)

Delete (x²-x+1)

so = (x-2)

0.2(x + 20) – 3 > –7 – 6.2x

Answers

Answer:

x > - 1.25

Step-by-step explanation:

0.2(x + 20) – 3 > –7 – 6.2x

0.2x + 4 - 3 > - 7 - 6.2x

0.2x + 1 > - 7 - 6.2x

Collect like terms

0.2x + 6.2x > -7 - 1

6.4x > -8

x > - 8/6.4

x > - 1.25

Note:

The > didn't change because you didn't divide by a negative value

Inequality signs changes when divided by a negative value

Answer:

Step-by-step explanation:

-1.25

What is the scale of the y-axis in this coordinate graph?


A. 1 tick mark represents 1 unit
B. 1 tick mark represents 8 units
C. 1 tick mark represents 12 units
D. 1 tick mark represents 16 units

Answers

Answer:

Obviously B

Step-by-step explanation:

4m square - 8mn+3n ( resolve into factors)​

Answers

Step-by-step explanation:

your answer will be ( 2m-3n)(2m-n)

Explanation is in the attachment

hope it is helpful to you

hope it is helpful to you

Step-by-step explanation:

[tex]4 {m}^{2} - 8mn + 3 {n}^{2} \\ 4 {m}^{2} - (2 + 6)mn + 3 {n}^{2} \\ 4 {m}^{2} - 2mn + 6mn + 3 {n}^{2} \\ 2m(2m - n) - 3n(2m - n) \\( 2m - 3n) \: (2m - n)[/tex]

Which operation will solve the following word problem? Jeff earns $14.00 per hour, Tom earns half as much as Jeff. How much does Tom earn per hour?


Multiplication


Subtraction


Addition


Division

Answers

Answer:

The correct option is (d).

Step-by-step explanation:

Given that,

Jeff earns $14.00 per hour.

Tom earns half as much as Jeff.

We need to find the amount earn by Tom per hour.

Tom's amount = Jeff's amount/2

So,

[tex]T=\dfrac{14}{2}\\\\T=\$7[/tex]

So, Tom earn $7 per hour. Hence, division operation is used. Jeff's amount is divided by 2.

Can someone help me with this math homework please!

Answers

Answer:

Step-by-step explanation:

In step one she has added 1x to each side of the statement

in step two she has taken five away from both sides of the equation therefore simplifying the entire equation

in step three she divided both sides by three to have the unknown value on one side of the equation

PLS HELP SOON WILL MARK BRAINLYEST

A railroad tunnel is shaped like a semi-ellipse, as shown below. A semiellipse is shown on the coordinate plane with vertices on the x axis and one point of intersection with the positive y axis. The height of the tunnel at the center is 35 ft, and the vertical clearance must be 21 ft at a point 8 ft from the center. Find an equation for the ellipse.

Answers

According to the question

b= 35 and (8,21) lies on the ellipse

After calculation we get a= 10

equation for the ellipse.

[tex] \frac{ {x}^{2} }{100} + \frac{ {y}^{2} }{1225} = 1[/tex]

find the domain of f(x)=sec(2x)

Answers

Answer:

*Refer the image attached

Step-by-step explanation:

*Refer the image attached

Hello, Brainly community!

This question is for all of those Calculus people out there.

The volume of a swimming pool is changing with respect to time, such that the volume is given by W(t), where W(t) is measured in cubic centimeters and t is measured in seconds. A tangent line is shown for W(t) at t = 3 seconds. Determine the best estimate for the value of the instantaneous rate of change of W(t) when t = 3.
(I've narrowed down the answer choices to 2, and just really need to find the right way of thinking to find the answer)

(A) W(lim t) as t goes to 3.
(B) [W(3.1) - W(2.9)] / 0.2.

Thank you in advance!

Answers

Answer:

(B)  [tex]\displaystyle \frac{W(3.1) - W(2.9)}{0.2}[/tex]

General Formulas and Concepts:

Calculus

Limits

Derivatives

The definition of a derivative is the slope of the tangent line.

Derivative Notation

Instantaneous Rates

Tangent Line: [tex]\displaystyle f'(x) = \frac{f(b) - f(a)}{b - a}[/tex]

Step-by-step explanation:

Since we are trying to find a rate at which W(t) changes, we must find the derivative at t = 3.

We are given 2 close answer choices that would have the same numerical answer but different meanings:

(A)  [tex]\displaystyle \lim_{t \to 3} W(t)[/tex](B)  [tex]\displaystyle \frac{W(3.1) - W(2.9)}{0.2}[/tex]

If we look at answer choice (A), we see that our units would simply just be volume. It would not have the units of a rate of change. Yes, it may be the closest numerically correct answer, but it does not tell us the rate at which the volume would be changing and it is not a derivative.

If we look at answer choice (B), we see that our units would be cm³/s, and that is most certainly a rate of change. Answer choice (B) is also a derivative at t = 3, and a derivative tells us what rate something is changing.

∴ Answer choice (B) will give us the best estimate for the value of the instantaneous rate of change of W(t) when t = 3.

Topic: AP Calculus AB/BC (Calculus I/I + II)

Unit: Differentiation

Book: College Calculus 10e

2 1/4 x 3 1/5 brainliest

Answers

Answer:

36/5

Step-by-step explanation:

9/4×16/5

144/20

36/5

hope this is helpful

Answer:

[tex]7\frac{1}{5}[/tex]

Step-by-step explanation:

1. start by turning the fractions improper fractions:

[tex]2\frac{1}{4} =\frac{9}{4}[/tex]

[tex]3\frac{1}{5} =\frac{16}{5}[/tex]

2. then multiply them together:

[tex]\frac{9}{4}[/tex] x [tex]\frac{16}{5}[/tex] = [tex]\frac{144}{20}[/tex]

3. then simplify the fraction:

[tex]\frac{144}{20}[/tex][tex]=\frac{36}{5}[/tex]

4. turn it into a proper fraction:

[tex]\frac{36}{5} =7\frac{1}{5}[/tex]

Tom goes to the book fair where paperback books are $3 and hardback books are $5.50. Travis
buys three paperback and four hardback books. How much change will Travis receive from a $50
bill
1.Equation/Expression:
2. Solve:

Answers

Answer:

1:  50 - (3(3) + 5.5(4))=x

2: okay so we know how much books are brought and how much each cost.

In this case paper backs are 3 dollars each and cost 3 dollars each. So 3 (3) is 9 dollars. The next scenario are hardbacks which are 5.50 each and Tom brought 4 of them. So that makes it 5.5(4) which is 22 dollars. So the total would be 9+22 which is 31 dollars. The question asks for change from a 50 dollar bill which you subtract and 50-31=19

The change would be 19 dollars

Can someone help me with this math homework please!

Answers

Answer:

1: the number of years since 2008 2. t is greater than or equal to 0 3. negative values 4. continuous

Step-by-step explanation:

1. t usually represents time

2. t must be greater than 0 as you can not go backward in time

3. the range must be positive as you can not have negative bobcats

4. its continuous because its a quadratic equation

1. no. of bobcats since 2008

2. greater than equal to 0

3 negative values

4. discrete because no. of bobcats cannot be broken into fraction.

PLISSSSSSSS HELPPPPP!!!!!!!!!!!!!!!!!!!!!
I will give brainliest............

Answers

I know the square root of sixteen is 4
1- 4
2- 1.5
3- 8
4- 1
5- 5
6- 9

The center of the circle is at the point
, and its radius is
units. The equation of this circle in standard form is
.

Answers

Is there a picture that goes with this?

The area of the rectangular sandbox at Dave's school is 117 square feet. The sandbox has a width of 9 feet as shown in the diagram. What is the perimeter of the sandbox?

Answers

Answer:

ay bru ima tell yu dhis rn is c

Step-by-step explanation:

Choose the smallest number 3 1/8 or 10/3

Answers

Answer:

10/3

Step-by-step explanation:

31/8= 3.8

10/3= 3.3

A cinema is doing a promotion to celebrate their 50th anniversary for 1 week. They give

away a free drink to every 98th customer, a free bag of popcorn to every 112th customer and

a free cinema ticket to every 224th customer. Which lucky customer will be the first to

receive all 3 items?​

Answers

Answer:

1,568 customer

Step-by-step explanation:

Find the lowest common multiple of 98, 112, and 224

98 = 98, 196, 294, 392, 490, 588, 686, 784, 882, 980, 1078, 1176, 1274, 1372, 1470, 1568, 1666

112 = 112, 224, 336, 448, 560, 672, 784, 896, 1008, 1120, 1232, 1344, 1456, 1568, 1680, 1792, 1904

224 = 224, 448, 672, 896, 1120, 1344, 1568, 1792, 2016, 2240

The lowest common multiple of 98, 112, and 224 is 1568

Therefore, the 1,568th customer will be the first to receive all 3 iitem

Find value of x.
A. 110
B. 47
C. 68
D. 112

Answers

Answer:

B

Step-by-step explanation:

The sum of the inner angles of a quadrilateral is 360 degrees

135 + 110 + 68 + x = 360

313 + x = 360

x = 47 degrees

Answer:

47

Step-by-step explanation:

whole thing is 360 degrees

68 + 110 + 135 = 313

360 - 313 = 47

x looks small too (if you had to guess in a multiple choice question)

According to the number line, what is the distance between points A and B?

0 6 units
7 units
O 12 units
O 14 units

Answers

Answer:

14 units

Step-by-step explanation:

A = - 2, B = 12

Therefore,

d(A, B) = 12 - (-2) = 12 + 2 = 14 units

Evaluate 2y when y = 6y

Answers

I believe the answer is 12y
im sure the answer is 12y

lidentify the domain of the function shown in the graph
O A 15257
O B. 19334
O C. 221
O D. All real numbers

Answers

Answer:

B.

Step-by-step explanation:

the visible line is the defined function.

this line goes from x=1 to x=4, and has the functional results from y=1 to y=7.

the domain is the valid interval of the input variable (typically x), while the range is the valid inescapable of the result variable (typically y).

so, B is the right answer.

If a bus travel for 120 minutes at a speed of 75 kilometers per hour how far has the bus traveled?

Answers

Answer:

150 km

Step-by-step explanation:

Put the minutes into hours 120min is 2 hours.

Distance = speed * time

Distance = 75 * 2

Distance = 150

Answer:

150 kilometers

Step-by-step explanation:

if the bus is going 75 kilometers an hour and they traveled for 120 minutes (exactly 2 hours) then you would just multiply 75 by 2 to get 150 kilometers total.

What’s the equation of the line that passes through the point (-4,4) and has a slope of 3/4

Answers

Answer:

y-y1=m(x-x1)

y-4=3/4(x+4)

y=3/4x+7

Other Questions
Look the phtoin the reply need 4 paragraphs in prg 1 : outline laras problem2:ask lara to think more carefully about the situation3: offer advice to make the friendship work4: tell him to speak to others about his problemplz plz helpp mee asap i will give more points and brainliest Flamenco is a Spanish word that means "flamingo". It refers to a type of dance. This dance has been compared to a flamingo because of the way the dancer moves. He or she has rapid, loud foot movements, and one foot is off ground almost all the time. What is the author's purpose in the passage? A. to entertain B. to explain something C. To persuade D. to express thoughts Question 1 of 10The triangles shown below may not be congruent.66V100100002017A. TrueB. FalseSUBMIT The vertices of a triangle are P(-6,1), Q(-2,-5) and R(8,1).Find the equation of the perpendicular bisector of the side QR please answer this What was one effect of the battles fought on the western frontier during the American Revolution? Josephine left home traveling at 25 mph. One hour later her friend, Steve, leaves from the same place and travels the same road traveling at 50 mph. How many hours will it take Steve to catch up to Josephine? A water pipe bursting in the winter is a result of which type of weathering?A. BiologicalB. ExfoliationC. ChemicalD. Freeze-thawHurry! Allen is an alcoholic who feels no guilt about his alcohol consumption. He regularly engages in reckless behavior, including getting into fights and spending money that he doesnt have. What else is likely true of Allen? Rohit thinks of a 4 digit number. The digit in the ones place is 3 more than the digit in the tens place, but 5 less than the digit in the thousands place. The value of the hundreds place is 600. The digit in thousands place is the greatest odd number. What is the number Rohit is thinking of? You are a prosecutor and are preparing for an upcoming trial. This is a case of arson, and the defendant is accused of setting fire to a barn, causing it to burn to the ground. You have an abundance of physical evidence that points to the defendant, but you also discover that a set of shoeprints found at the scene matches another suspect. This would be an example of what type of evidence? The length of a rectangle is twice its width the perimiter is 60 ft find its area The amount of tax on a chair was $3.60. The tax rate was 5%. Find the original price of the chair.Bianca solved the problem below. Find Biancas error.0.05(3.60) = Original priceThe original price is $0.18. Which shape has the greatest number of lines of symmetry?A. rhombusB. squareC. rectangleD. parallelogram The x intercepts of the function f(x) = 2x(x-5)^2(x+4)^3are Can Someone Help Me With This ? Which of the followingstatement is true accordingto the graphic in regards tothe spread of Islam?A. Islam spread into Africa and Europe.B. Islam was isolated in the Saudi Arabianpeninsula.C. The spread of Islam only occurred from622-632 CE.D. The spread of Islam began in WestAfrica first. A designer needs to select the material for a plate under tensile stress. Assuming that the applied tensile force is 13,000 lb and the area under the stress is 4 square inches, determine which material should be selected to assure safety. Assume safety factor is 2. Material A: Ultimate Tensile stress is 8000 lb/in2Material B: Ultimate Tensile stress is 5500 lb/in2 Solve using the substitution method 16x 4y = 16 4x - 4 = y What process adds carbon dioxide to the air?O nitrogen cycleO water cycleO respirationO photosynthesis